Home   »   Bihar Police SI Answer Key 2026
Top Performing

Bihar Police SI Answer Key 2026: Memory Based Question Paper with Answer and Explanaion

The Bihar Police SI Answer Key 2026 is one of the most awaited updates for candidates who appeared in the BPSSC Sub Inspector (SI) Preliminary Examination. The answer key helps aspirants check their responses, calculate their estimated scores, and predict their chances of qualifying for the Mains exam.

We are providing the Bihar Police SI Memory-Based Question Paper 2026 (January 18, Shift 1) in English, including:

  • Complete Questions
  • Multiple Choice Options
  • Correct Answers
  • Detailed Explanations

This memory-based paper is extremely useful for candidates who want to analyze the exam trend and prepare effectively for upcoming shifts and future exams.

Bihar Police SI Answer Key 2026 – Official Release Date

The exact date for the release of the Bihar Police SI Answer Key 2026 has not been declared by BPSSC yet. However, going by last year’s trend, it is anticipated that it will be released in the first week of February 2026. Once the answers are out, the candidates will be able to download the PDF file containing the correct answers, map their own answers to the correct ones, assess their estimated scores, and send objections for the incorrect answers marked, if needed.

Particulars Details
Exam Name Bihar Police SI Exam 2026
Conducting Body BPSSC
Post Sub Inspector (SI)
Vacancies 1799
Exam Dates 18 & 21 January 2026
Shift Shift 1 & Shift 2
Selection Process Prelims → Mains → Interview
Answer Key Mode PDF (Online)

Bihar Police SI Memory-Based Question Paper 2026

Below are some memory-based questions with options and explanations:

Q.1 The oath of office to the Chief Minister of a State is administered by whom?

Options:

  • A. President of India
  • B. Governor of the State
  • C. Chief Justice of India
  • D. Speaker of the Legislative Assembly

Answer: B. Governor of the State

Explanation: According to Article 164 of the Indian Constitution, the Chief Minister is appointed by the Governor. Before entering office, the Governor administers the oaths of office and secrecy to the Chief Minister. The Governor acts as the constitutional head of the state.

Information Booster:

  • The forms of oath are mentioned in the Third Schedule of the Constitution
  • The Chief Minister holds office during the pleasure of the Governor, though practically as long as they enjoy a majority in the assembly

Q.2 In which of the following cities was the Sepak Takraw 2025 World Cup (ISTAF) held in India in March 2025?

Options:

  • A. Mumbai
  • B. Patna
  • C. Bengaluru
  • D. New Delhi

Answer: B. Patna

Explanation: The Sepak Takraw World Cup 2025 was hosted in Patna, Bihar. This event marked a significant milestone for sports in Bihar, bringing international attention to the city.

Information Booster:

  • Sepak Takraw is a sport native to Southeast Asia, often described as “kick volleyball”
  • Players use a rattan ball and can only touch it with their feet, knees, head, and chest
  • The event was organized by the International Sepak Takraw Federation (ISTAF)

Q.3 Which of the following sessions of Indian National Congress was presided over by Mahatma Gandhi?

Options:

  • A. 1922; Gaya
  • B. 1923; Delhi
  • C. 1924; Belgaum
  • D. 1925; Kanpur

Answer: C. 1924; Belgaum

Explanation: The 1924 Belgaum session of the Indian National Congress was presided over by Mahatma Gandhi. It was the only session where he served as the president of the Congress, focusing on unity and Swaraj (self-rule).

Key Points:

  1. The session emphasized the promotion of Khadi and non-cooperation
  2. Gandhi’s leadership symbolized moral and spiritual commitment to freedom
  3. Resolutions included a focus on communal harmony and rural development
  4. Marked a shift towards mass mobilization in the independence struggle
  5. Belgaum, located in Karnataka, became a historic site for this event

Knowledge Booster:

  • 1922, Gaya: Focused on Swaraj post the Chauri Chaura incident
  • 1923, Delhi: Known for the split between Swarajists and No-Changers
  • 1925, Kanpur: Sarojini Naidu became the first woman to preside over the Congress

Q.4 Nepal shares its borders with which of the following Indian states?

Options:

  • A. West Bengal
  • B. Mizoram
  • C. Rajasthan
  • D. Punjab

Answer: A. West Bengal

Explanation: West Bengal shares a significant border with Nepal in its northern region, specifically along the Darjeeling and Kalimpong districts. This border is crucial as it connects Nepal to the “Chicken’s Neck” (Siliguri Corridor), which is a vital trade and transit route.

Information Booster:

  • Nepal is a landlocked country surrounded by India on three sides (East, South, and West) and China on the North
  • India shares a border of approximately 1,751 km with Nepal
  • A total of 5 Indian states share a border with Nepal: Uttarakhand, Uttar Pradesh, Bihar, West Bengal, and Sikkim
  • The border is open and porous, governed by the 1950 Indo-Nepal Treaty of Peace and Friendship

Q.5 Which article deals with the abolition of untouchability?

Options:

  • A. Article 14
  • B. Article 15
  • C. Article 17
  • D. Article 21

Answer: C. Article 17

Explanation: Article 17 of the Indian Constitution abolishes ‘Untouchability’ and forbids its practice in any form. The enforcement of any disability arising out of untouchability is an offense punishable in accordance with the law.

Key Points:

  • It is a fundamental right provided under the ‘Right to Equality’ (Articles 14-18)
  • To give effect to this article, the Untouchability (Offences) Act, 1955 was enacted, later renamed as the Civil Rights Protection Act
  • This article is absolute, meaning it has no exceptions

Information Booster:

  • The term ‘Untouchability’ is not defined in the Constitution or the Act
  • It is one of the few fundamental rights available against private individuals as well as the state

Additional Knowledge:

  • Article 14 (Option A): Deals with equality before the law
  • Article 15 (Option B): Prohibits discrimination on grounds of religion, race, caste, sex, or place of birth
  • Article 21 (Option D): Deals with the protection of life and personal liberty

Q.6 The Indian Ocean is bounded by _____ in the east.

Options:

  • A. Antarctica
  • B. Australia
  • C. Africa
  • D. Iran

Answer: B. Australia

Explanation: The Indian Ocean is bounded by the continent of Australia and the Sunda Islands (Indonesia) to the east. This eastern boundary separates the Indian Ocean from the Pacific Ocean.

Key Points:

  • Geographically, the ocean is enclosed by Asia to the north, Africa to the west, and Australia to the east

Information Booster:

  • It is the third-largest ocean in the world, covering approximately 20% of the Earth’s water surface
  • It is the only ocean named after a country (India)
  • The deepest point in the Indian Ocean is the Sunda Trench (also known as the Java Trench)

Additional Knowledge:

  • (A) Antarctica: Located to the south, forms the southern boundary
  • (C) Africa: Forms the western boundary
  • (D) Iran: Located to the north; borders the Arabian Sea, a northern extension of the Indian Ocean

Q.7 Citric acid is produced by

Options:

  • A. Aspergillus niger
  • B. Streptococcus
  • C. Acetobacter suboxydans
  • D. Candida utilis

Answer: A. Aspergillus niger

Explanation: Citric acid is industrially produced using the fungus Aspergillus niger through fermentation of sugars. The microorganism efficiently converts carbohydrates into citric acid, which is widely used in food, pharmaceuticals, and cleaning products.

Additional Information:

  • Option (B) Streptococcus: Produces lactic acid
  • Option (C) Acetobacter: Oxidizes ethanol to acetic acid
  • Option (D) Candida utilis: Used as single-cell protein

Q.8 Who among the following invited Mahatma Gandhi to Champaran?

Options:

  • A. Rajendra Prasad
  • B. Raj Kumar Shukla
  • C. J.B. Kripalani
  • D. Mazhar-ul-Haq

Answer: B. Raj Kumar Shukla

Explanation: The Champaran Satyagraha of 1917 was Mahatma Gandhi’s first major application of the Satyagraha (non-violent resistance) technique in India and marked his true entry into Indian national politics. The movement addressed the plight of poor peasants in the Champaran district of Bihar.

Key Points:

  • Peasants were forced by British indigo planters to cultivate indigo on a mandatory portion of their land (usually 3/20th of their holding, a system called Tinkathia)
  • They were forced to sell the indigo at fixed, low prices, leading to severe exploitation and poverty

Information Booster:

  • Raj Kumar Shukla was a persistent local farmer/peasant from Champaran
  • Shukla attended the 31st Session of the Indian National Congress in Lucknow in December 1916
  • He repeatedly approached Congress leaders, including Gandhi, narrating the miseries of the Tinkathia system
  • Shukla’s persistence eventually convinced Gandhi to visit Champaran
  • Gandhi arrived in Champaran in April 1917, accompanied by Shukla

Additional Knowledge:

  • Rajendra Prasad: Prominent lawyer from Bihar who joined Gandhi’s team and later became India’s first President
  • J.B. Kripalani: Acharya J.B. Kripalani was a history professor who met Gandhi upon his arrival and provided initial support
  • Mazhar-ul-Haq: Distinguished lawyer and nationalist leader who actively participated in the Champaran movement

Q.9 Through which of the following passes does the Jawahar Tunnel pass?

Options:

  • A. Rohtang
  • B. Nathula
  • C. Banihal
  • D. Dungri La

Answer: C. Banihal

Explanation: The Jawahar Tunnel, which connects Jammu and Kashmir with the rest of India, passes through the Banihal Pass. This tunnel plays a crucial role in maintaining year-round connectivity to the region, as it bypasses the difficult mountain terrain.

Information Booster:

  • The Jawahar Tunnel is a vital part of the National Highway 1A
  • It is located at the Banihal Pass in the Pir Panjal Range of the Himalayas
  • Named after Jawaharlal Nehru
  • Strategically important, especially during winter months when heavy snowfalls make other mountain passes difficult or impassable

Additional Information:

  • (A) Rohtang: Located in Himachal Pradesh, does not pass through the Jawahar Tunnel
  • (B) Nathula: Located in Sikkim; serves as an important border pass between India and China
  • (D) Dungri La: Not a pass through which the Jawahar Tunnel passes

Q.10 Translate the following Hindi sentence into English: “बहुत कम लोग सच्चाई को स्वीकार करते हैं।”

Options:

  • A. Very few people accepted the truth.
  • B. Few people accept the truth.
  • C. Very few people accept the truth.
  • D. Only people accept the truth.

Answer: C. Very few people accept the truth.

Explanation: The sentence is in the Simple Present Tense and expresses a general fact.

Detailed Explanation:

  • The phrase “बहुत कम लोग” means “very few people,” emphasizing scarcity
  • Option (C) correctly uses “very few,” which conveys the intended negative sense
  • The verb “accept” correctly matches the plural subject “people”
  • The meaning and grammatical structure are accurately preserved

Why Other Options Are Incorrect:

  • (A) uses past tense
  • (B) changes the degree of scarcity
  • (D) is grammatically incorrect and meaningless

Q.11 Which time-period belongs to the Khalji dynasty?

Options:

  • A. 1206–1236
  • B. 1290–1320
  • C. 1320–1414
  • D. 1414–1451

Answer: B. 1290–1320

Explanation: The Khalji dynasty was the second dynasty to rule the Delhi Sultanate. It was founded by Jalal-ud-din Khalji in 1290. The dynasty ended in 1320 when Khusrau Khan was defeated by Ghiyas-ud-din Tughlaq.

Information Booster:

  • Alauddin Khalji is known for his market reforms and administrative measures
  • The Khalji dynasty was followed by the Tughlaq dynasty

Additional Knowledge:

  • 1206–1290 (Option A context): Period of the Slave (Mamluk) Dynasty
  • 1320–1414 (Option C): Period of the Tughlaq Dynasty
  • 1414–1451 (Option D): Period of the Sayyid Dynasty

Q.12 Consider the following statements: 1. The Supreme Court of India can issue writs only for enforcement of Fundamental Rights. 2. High Courts can issue writs for both Fundamental Rights and other legal rights. 3. SC inaugural session was held on 26 January 1950.

Options:

  • A. 2 and 3 only
  • B. 1 and 2 only
  • C. 1 and 3 only
  • D. 1, 2 and 3

Answer: B. 1 and 2 only

Explanation:

  • Article 32 empowers SC to issue writs for Fundamental Rights only
  • Article 226 lets High Courts issue writs for FRs and other legal rights (“for any other purpose”)
  • The Supreme Court was inaugurated on 28 January 1950 (not 26 Jan)
  • Hence 1 & 2 true; 3 false

Information Booster:

  • Five writs: Habeas Corpus, Mandamus, Prohibition, Certiorari, Quo Warranto
  • SC succeeded the Federal Court (1937–1950)
  • First CJI: H. J. Kania
  • Art. 136: SC’s Special Leave Petition power
  • Writs are speedy remedies against state action

Additional Knowledge:

  • High Courts’ writ jurisdiction is wider than SC’s
  • 26 Jan 1950 = Constitution came into force; SC started two days later

Q.13 Who has been appointed as the Chief Information Commissioner (CIC) of India in December 2025?

Options:

  • A. Heeralal Samariya
  • B. Raj Kumar Goyal
  • C. Jaya Verma Sinha
  • D. Vinod Kumar Tiwari

Answer: B. Raj Kumar Goyal

Explanation: Former Justice Secretary Raj Kumar Goyal took oath as the Chief Information Commissioner in December 2025. He filled the vacancy created after the demitting of office by Heeralal Samariya.

Key Points:

  • His appointment strengthens the transparency framework under the RTI Act, 2005

Information Booster:

  • The CIC heads the Central Information Commission
  • The CIC plays a crucial role in adjudicating RTI appeals and complaints

Additional Knowledge:

  • (A) Heeralal Samariya: Former Chief Information Commissioner who demitted office in September 2025
  • (C) Jaya Verma Sinha: Former Railway Board Chairperson; recommended as Information Commissioner, not CIC

Q.14 Which Amendment added the word ‘secular’ to the Preamble of the Indian Constitution?

Options:

  • A. 41st Amendment
  • B. 42nd Amendment
  • C. 43rd Amendment
  • D. 44th Amendment

Answer: B. 42nd Amendment

Explanation: The word ‘secular’ was added to the Preamble of the Indian Constitution through the 42nd Amendment in 1976. The 42nd Amendment of 1976, also known as the Constitution (Forty-Second Amendment) Act, was one of the most comprehensive amendments to the Indian Constitution. It was enacted during the Emergency period under Prime Minister Indira Gandhi’s leadership.

Key Points:

  • This amendment not only added the word ‘secular’ but also made changes to the Preamble by emphasizing that India is not only a sovereign and democratic republic but also secular and socialist
  • The addition of the word ‘secular’ signified that the Indian state does not favor any religion and treats all religions equally
  • This reaffirms the principle of religious tolerance and freedom of religion guaranteed under Article 25 of the Constitution
  • The Preamble now reads: “We, the people of India, having solemnly resolved to constitute India into a Sovereign, Socialist, Secular, Democratic Republic…”

Information Booster:

  • The 42nd Amendment was passed during the Emergency period (1975-1977)
  • The term ‘Secular’ means that the government does not recognize any religion as the state religion and ensures that all religions are treated equally
  • This amendment was later partially reversed by the 44th Amendment of 1978
  • The concept of secularism in India is not just legal but also cultural

Additional Information (Other Options):

  • Option (A) – 41st Amendment (1975): Dealt with various issues but did not introduce the word ‘secular’
  • Option (C) – 43rd Amendment (1977): Reversed some provisions of the 42nd Amendment
  • Option (D) – 44th Amendment (1978): Focused on restoring individual freedoms and curbing the powers of the President

Q.15 When was the Jan Dhan Yojana launched?

Options:

  • A. 2015
  • B. 2016
  • C. 2013
  • D. 2014

Answer: D. 2014

Explanation: Jan Dhan Yojana was launched by Prime Minister Narendra Modi on August 28, 2014. It aims to provide universal access to banking facilities and ensure that every Indian has a bank account.

Key Points:

  • The scheme also focuses on promoting financial literacy, digital banking, and insurance coverage for all
  • The scheme aims to bring the unbanked population under the financial system
  • It provides financial services like bank accounts, insurance, pension, and credit
  • It encourages people to save money and avail themselves of overdraft facilities

Q.16 When did the Chuar Rebellion start?

Options:

  • A. 1771 AD
  • B. 1772 AD
  • C. 1871 AD
  • D. 1872 AD

Answer: A. 1771 AD

Explanation: The Chuar Rebellion began in 1771 in the Jungle Mahals region (now parts of West Bengal and Jharkhand), where tribal peasants, called Chuars, revolted against oppressive taxes and land encroachments by the British East India Company.

Key Points:

  • This rebellion was one of the earliest tribal uprisings against British colonial authority
  • It arose from economic exploitation and socio-political oppression

Information Booster:

  • The Chuars employed guerrilla tactics, attacking British officials and disrupting revenue collection
  • The rebellion exposed the discontent among tribal communities over loss of traditional rights and livelihoods under colonial rule

Q.17 Which country is set to host the FIDE Chess World Cup 2025?

Options:

  • A. Russia
  • B. India
  • C. Norway
  • D. United States

Answer: B. India

Explanation: India will host the prestigious FIDE Chess World Cup 2025 from October 31 to November 27, 2025. This marks the first major international chess tournament hosted by India since the 2022 Chennai Chess Olympiad.

Key Points:

  • The tournament will feature over 200 players from around the world
  • Follows a knockout format
  • Offers three qualification spots for the Candidates Tournament, which determines the challenger for the World Chess Championship

Information Booster:

  • Gukesh Dommaraju is the current world chess champion and the youngest person to ever hold the title. He defeated China’s Ding Liren in the 2024 World Chess Championship
  • R. Praggnanandhaa: Won the silver medal in the 2023 FIDE Chess World Cup
  • Viswanathan Anand: The only Indian to win the FIDE World Cup, clinching titles in 2000 and 2002

Facts About Chess:

  • Origin: Chess originated in India during the 6th century and was called Chaturanga
  • First World Champion: Wilhelm Steinitz (1886)
  • Indian Grandmasters: India has over 80 Grandmasters

Q.18 Which one of the following pairs of Dynasty and King of the Vijayanagara Empire is not correctly matched?

Options:

  • A. Sangama Dynasty – Harihara II
  • B. Saluva Dynasty – Timma
  • C. Aravidu Dynasty – Tirumala
  • D. Tuluva Dynasty – Gunda

Answer: D. Tuluva Dynasty – Gunda

Explanation: The Tuluva Dynasty was founded by Vira Narasimha Raya. Its most famous ruler was Krishna Deva Raya. Gunda was not a ruler of the Tuluva dynasty, and no major king by this name is documented in Vijayanagara history. Hence, this pair is incorrectly matched.

Information Booster:

  • The Vijayanagara Empire had four major dynasties: Sangama, Saluva, Tuluva, and Aravidu
  • It was one of the most powerful South Indian empires, known for administrative reforms and temple architecture

Additional Knowledge:

  • (A) Sangama Dynasty – Harihara II: Ruled after Bukka Raya I and was a notable king who expanded the empire
  • (B) Saluva Dynasty – Timma: Timma (Thimma Bhupala) was the son of Saluva Narasimha; briefly ruled before the rise of Tuluva Narasa Nayaka
  • (C) Aravidu Dynasty – Tirumala: Tirumala Deva Raya was the founder of the Aravidu dynasty; ruled after the decline of the Tuluva line post-Battle of Talikota (1565)

Q.19 Which of the following statements regarding the Freedom Struggle from 1920 to 1935 is/are correct?

  1. In Bihar and Bengal, the Non-Cooperation Movement owed its effectiveness to the participation of peasants.
  2. In Bihar they were organised against the planters under the banner of Kisan Sabha.
  3. In Midnapore in Bengal, Mahishya peasants rallied against the taxes of the Union Board under the leadership of Birendranath Sasmal.

Options:

  • A. 1, 2 and 3
  • B. 2 and 3 only
  • C. 1 only
  • D. 3 only

Answer: A. 1, 2 and 3

Explanation: All three statements are correct:

  • Peasant involvement in Bihar and Bengal greatly enhanced the reach and effectiveness of the Non-Cooperation and related movements
  • In Bihar, peasants mobilised under organisations like the Kisan Sabha against planters and oppressive zamindars
  • In Midnapore (Bengal), Mahishya peasants, led by figures like Birendranath Sasmal, resisted Union Board taxes
  • Local grievances were linked to the broader national movement

Information Booster:

  • The 1920–35 period saw multiple, regionally specific peasant revolts that fed into the national struggle
  • Leaders such as Swami Sahajanand Saraswati strengthened organisational structures for peasants

Additional Knowledge:

  • Statement 1: Highlights the pivotal role of peasants in non-cooperation outcomes
  • Statement 2: Kisan Sabhas were important in Bihar’s peasant agitation against planters
  • Statement 3: Birendranath Sasmal’s leadership in Midnapore is well documented in regional accounts

Q.20 Which country has decided to include YouTube in its social media ban for children under 16?

Options:

  • A. United States
  • B. United Kingdom
  • C. Australia
  • D. Canada

Answer: C. Australia

Explanation: Australia has decided to include YouTube in its social media ban for children under 16, effective December 2025. The decision was made to reduce children’s exposure to harmful content on the platform, as reported by the eSafety Commission.

Information Booster:

  • The ban will also apply to other platforms like Facebook, Instagram, TikTok, Snapchat, and X (formerly Twitter)
  • YouTube Kids remains exempt from the ban, as it is a safer platform with limited functionality

Additional Information:

  • Critics of the ban argue that it may limit access for vulnerable or isolated children who rely on online communities
  • However, the government believes it’s necessary to protect children from harmful digital content

Q.21 What is the compounded ratio of (1 : 3), (5 : 11) and (22 : 25)?

Options:

  • A. 1 : 25
  • B. 3 : 5
  • C. 11 : 25
  • D. 2 : 15

Answer: D. 2 : 15

Explanation: Formula Used: Compounded ratio = Product of antecedents : Product of consequents

Solution:

  • Antecedents = 1 × 5 × 22 = 110
  • Consequents = 3 × 11 × 25 = 825
  • Ratio = 110 : 825 = 2 : 15

Final Answer: 2 : 15

Q.22 Identify the following constitutional attributes with their respective democratic values:

Column A:

  • a) Fundamental Duties
  • b) Equal voting rights
  • c) Local self-government provisions
  • d) Religious freedom

Column B:

  • i. Grassroots democracy
  • ii. Respect for individual conscience
  • iii. Recognition of citizens’ responsibilities
  • iv. Principle of universal franchise

Options:

  • A. a-ii, b-i, c-iv, d-iii
  • B. a-i, b-ii, c-iii, d-iv
  • C. a-iii, b-iv, c-i, d-ii
  • D. a-iv, b-iii, c-ii, d-i

Answer: C. a-iii, b-iv, c-i, d-ii

Explanation:

  • Fundamental Duties → Recognition of citizens’ responsibilities (iii)
  • Equal voting rights → Principle of universal franchise (iv)
  • Local self-government provisions → Grassroots democracy (i)
  • Religious freedom → Respect for individual conscience (ii)

Information Booster:

  • Fundamental Duties → Added by 42nd Amendment, 1976 (Part IV-A, Article 51A)
  • Universal Adult Franchise → Article 326 (equality in voting rights)
  • Local self-government → 73rd & 74th Amendments (1992) gave constitutional status to Panchayati Raj & Municipalities
  • Religious Freedom → Articles 25–28 guarantee freedom of religion in India
  • Together, these reflect the democratic and secular ethos of the Constitution

Q.23 The volume (in cm³) of a wire of diameter 10 cm and length 56 m is: (take π = 22/7)

Options:

  • A. 441000
  • B. 440700
  • C. 440000
  • D. 440400

Answer: C. 440000

Explanation: Given:

  • Diameter of the wire = 10 cm
  • Radius r = 10/2 = 5 cm
  • Length of the wire = 56 m = 5600 cm
  • π = 22/7

Formula Used: Volume of a cylinder = πr²h

Solution:

  • Volume = (22/7) × 5² × 5600
  • Volume = (22/7) × 25 × 5600
  • Volume = 22 × 25 × 800
  • Volume = 440000 cm³

Q.24 If P is 30% more than Q, and R is 25% more than P, then what is Q : R?

Options:

  • A. 4 : 5
  • B. 8 : 13
  • C. 5 : 4
  • D. 33 : 20

Answer: B. 8 : 13

Explanation: Given:

  • P = 30% more than Q
  • R = 25% more than P

Formula Used: New value = (1 + x/100) × Original value

Solution:

  • P = 130% of Q = (130/100)Q = (13/10)Q
  • R = 125% of P = (125/100)P = (5/4)P
  • R = (5/4) × (13/10)Q = (65/40)Q
  • Q : R = Q : (65/40)Q = 40 : 65 = 8 : 13

Final Answer: 8 : 13

Q.25 Article 323 of the Indian Constitution deals with _____________

Options:

  • A. expenses of Public Services Commissions
  • B. reports of Public Services Commissions
  • C. functions of Public Services Commissions
  • D. power to extend functions of Public Services Commissions

Answer: B. reports of Public Services Commissions

Explanation: Article 323 of the Indian Constitution mandates that each Public Service Commission must present an annual report to the President or the Governor, as the case may be, regarding the work done by the Commission.

Key Points:

  • This article ensures transparency and accountability in the functioning of the Union Public Service Commission (UPSC) and State Public Service Commissions
  • The reports include details on the commission’s activities, including examinations conducted, recruitments made, and other relevant functions

Information Booster:

  • Article 322: Relates to the expenses of Public Service Commissions
  • Article 323A: Provides for the establishment of Administrative Tribunals
  • Article 323B: Deals with the establishment of Tribunals for other matters

Q.26 Which of the following is NOT correctly matched?

Options:

  • A. Lushai Revolt – Assam
  • B. Heraka Movement – West Bengal
  • C. Kol Rebellion – Jharkhand
  • D. Ramosi Rebellion – Maharashtra

Answer: B. Heraka Movement – West Bengal

Explanation: The Heraka Movement was a socio-religious reform movement of the Zeliangrong tribes in Assam, Manipur, and Nagaland, not West Bengal. It was associated with Rani Gaidinliu and aimed to revive traditional Naga religion.

Information Booster:

  • Lushai Revolt (1890s) occurred in Assam–Mizoram region
  • Kol Rebellion (1831–32) took place in Chotanagpur (present Jharkhand)
  • Ramosi Rebellion (1822 & 1827) happened under Chittur Singh in Maharashtra
  • Heraka promoted monotheism among Zeliangrong tribes
  • Rani Gaidinliu was jailed by the British for her role in the movement
  • Rani Gaidinliu received Padma Bhushan (1982) for her contribution

Q.27 The Green Revolution started in the year 1965 and the _____________ five year plan was between 1961-1966.

Options:

  • A. 5th
  • B. 2nd
  • C. 1st
  • D. 3rd

Answer: D. 3rd

Explanation: The Green Revolution in India began in 1965, during the Third Five Year Plan (1961-1966). The Third Five Year Plan focused on agriculture and improving food production in India, but it was the Green Revolution that significantly boosted India’s agricultural output, particularly in the production of wheat and rice.

Information Booster:

  • Green Revolution: Introduced high-yielding varieties (HYVs) of seeds, especially for wheat and rice, along with the use of chemical fertilizers, pesticides, and advanced irrigation techniques
  • The Green Revolution helped India become self-sufficient in food production
  • Third Five Year Plan: Aimed at making India self-reliant and focused on agriculture, education, and heavy industries
  • Due to various issues such as the Indo-China war and poor monsoon, the agricultural sector needed more efforts, which led to the introduction of the Green Revolution

Q.28 Which of the following Articles of the Indian Constitution provides for an independent office of the Comptroller and Auditor General (CAG) of India?

Options:

  • A. Article 145
  • B. Article 146
  • C. Article 147
  • D. Article 148

Answer: D. Article 148

Explanation: Article 148 of the Indian Constitution provides for an independent office of the Comptroller and Auditor General (CAG) of India.

Key Points:

  • The CAG is the head of the Indian Audit and Accounts Department
  • He is the guardian of the public purse and controls the entire financial system of the country at both levels—the Centre and the states
  • He is appointed by the President of India by a warrant under his hand and seal
  • The CAG holds office for a term of six years or until he attains the age of 65 years, whichever is earlier
  • He can be removed by the President on the same grounds and in the same manner as a judge of the Supreme Court

Additional Knowledge:

  • Article 145 (Option A): Deals with Rules of Court
  • Article 146 (Option B): Deals with officers and servants and the expenses of the Supreme Court
  • Article 147 (Option C): Deals with the interpretation of the Constitution

Q.29 The catalyst used for the decomposition of potassium chlorate {KClO₃} is:

Options:

  • A. ZnO
  • B. MnO₂
  • C. CuO
  • D. K₂O

Answer: B. MnO₂

Explanation: Potassium chlorate decomposes on heating to produce potassium chloride and oxygen gas. This decomposition occurs much more rapidly in the presence of manganese dioxide, which acts as a catalyst. MnO₂ lowers the activation energy of the reaction without undergoing any permanent chemical change itself.

Reaction: 2KClO₃ → 2KCl + 3O₂ (in presence of MnO₂)

Thus, manganese dioxide is commonly used as a catalyst for this reaction.

Additional Information:

  • Option (A) is incorrect because zinc oxide does not effectively catalyse the decomposition of potassium chlorate
  • Option (C) is incorrect because copper oxide is not used as a catalyst in this reaction
  • Option (D) is incorrect because potassium oxide does not play any catalytic role in the decomposition of KClO₃

Q.30 What is the main difference between prokaryotic and eukaryotic cells?

Options:

  • A. Prokaryotic cells have a nucleus, while eukaryotic cells do not
  • B. Eukaryotic cells lack membrane-bound organelles, unlike prokaryotic cells
  • C. Prokaryotic cells lack a true nucleus, while eukaryotic cells have a membrane-bound nucleus
  • D. Eukaryotic cells are unicellular, and prokaryotic cells are multicellular

Answer: C. Prokaryotic cells lack a true nucleus, while eukaryotic cells have a membrane-bound nucleus

Explanation: The main difference is that prokaryotic cells lack a true nucleus, while eukaryotic cells have a membrane-bound nucleus.

Information Booster:

  • Prokaryotic cells have a simple structure without membrane-bound organelles, and their DNA is found in the nucleoid
  • Eukaryotic cells are more complex and contain membrane-bound organelles like the nucleus, mitochondria, and Golgi apparatus
  • Prokaryotic cells are usually unicellular organisms (e.g., bacteria), while eukaryotic cells can be either unicellular or multicellular

Q.31 Who was sworn in as the Chief Minister of Bihar for a record 10th time in November 2025?

Options:

  • A. Samrat Choudhary
  • B. Tejashwi Yadav
  • C. Vijay Kumar Sinha
  • D. Nitish Kumar

Answer: D. Nitish Kumar

Explanation: Nitish Kumar, President of JD(U), was sworn in as the Chief Minister of Bihar for the 10th time after the NDA’s victory in the 2025 Bihar Assembly Elections.

Information Booster:

  • Oath Ceremony Venue: Gandhi Maidan, Patna
  • Oath Administered By: Bihar Governor Arif Mohammad Khan
  • Deputy Chief Ministers: Samrat Choudhary and Vijay Kumar Sinha
  • Council of Ministers: 27 (including CM)
  • Election Outcome: NDA won 202 out of 243 seats; MGB secured 35 seats
  • Youngest MLA: Maithili Thakur (Age 25), representing Alinagar
  • Alliance Partners: JD(U), BJP, LJP, HAM (Secular), RLM

Q.32 Where was India’s First National Dolphin Research Centre (NDRC) inaugurated?

Options:

  • A. Varanasi, Uttar Pradesh
  • B. Patna, Bihar
  • C. Guwahati, Assam
  • D. Kolkata, West Bengal

Answer: B. Patna, Bihar

Explanation: India’s first National Dolphin Research Centre (NDRC) was inaugurated in Patna, Bihar, marking a major milestone in the conservation of the Gangetic dolphin. This center is the first of its kind in Asia, dedicated to the study and protection of this endangered species. The NDRC will serve as a research hub, helping scientists monitor and conserve river dolphins while also promoting awareness about the importance of freshwater biodiversity.

About the Gangetic Dolphin:

  • The Gangetic dolphin (Platanista gangetica) is India’s National Aquatic Animal
  • Classified as Endangered by the IUCN Red List
  • Protected under Schedule I of the Wildlife (Protection) Act, 1972
  • The dolphin is blind and uses echolocation to navigate and hunt in murky waters

Information Booster:

  • Location & Purpose of NDRC: Situated in Patna, Bihar, near the Ganga River; focuses on conservation, research, and habitat protection
  • Found in: Ganges, Brahmaputra, and Meghna river systems (India, Nepal, and Bangladesh)
  • Gives birth once every 2-3 years, making conservation efforts crucial

Additional Knowledge:

  • IUCN Status: Endangered
  • Wildlife (Protection) Act, 1972: Schedule I species (highest protection)
  • CITES: Appendix I (bans international trade)
  • Project Dolphin (2021): Aims to enhance dolphin conservation
  • Vikramshila Dolphin Sanctuary (Bihar): Key habitat for Gangetic dolphins

Q.33 Baking powder is a mixture of:

Options:

  • A. sodium carbonate and tartaric acid
  • B. baking soda and washing soda
  • C. baking soda and bleaching powder
  • D. baking soda and mild edible acid

Answer: D. baking soda and mild edible acid

Explanation: Baking powder is a mixture of the following components:

  1. Sodium bicarbonate (NaHCO₃): Also known as baking soda, it serves as the base and releases carbon dioxide when it reacts
  2. Tartaric acid: An acidic component that reacts with sodium bicarbonate to release carbon dioxide
  3. Starch: Added as a filler to absorb moisture and prevent premature reactions between the acid and the base

Q.34 The term ‘green revolution’ was coined by:

Options:

  • A. N.E. Borlaug
  • B. R.N. Singh
  • C. William S. Gaud
  • D. M.S. Swaminathan

Answer: C. William S. Gaud

Explanation: William S Gaud is a former administrator of the United States Agency for international development. He coined the term ‘green revolution’ first time.

Key Information:

  • The green revolution is the initiative launched for the development of new crop varieties to increase agricultural production in developing countries
  • Norman Borlaug is considered the “father of the green revolution”
  • M S Swaminathan is considered as the “father of the green revolution in India”
  • The term “evergreen revolution” was given by M S Swaminathan
  • He was the chairman of the national commission on farmers in India
  • The green revolution was started its initial phases in Mexico
  • Punjab pioneered the green revolution in India

Q.35 What is the capital of Colombia?

Options:

  • A. Caracas
  • B. Bogotá
  • C. Lima
  • D. Santiago

Answer: B. Bogotá

Explanation: Bogotá is the capital city of Colombia, known for its cultural and political significance in the country.

Q.36 Which dynasty was founded after the Maurya dynasty?

Options:

  • A. Shaka dynasty
  • B. Kushan dynasty
  • C. Satavahana dynasty
  • D. Shunga dynasty

Answer: D. Shunga dynasty

Explanation: The Shunga dynasty was formed immediately following the decline of the Maurya Empire. After the assassination of the last Mauryan ruler, Brihadratha, in 185 BCE, his military commander Pushyamitra Shunga assumed power and established the Shunga dynasty.

Key Points:

  • This transition marked a shift from Mauryan Buddhist influence to a renewed emphasis on Brahmanical practices
  • The Shungas maintained control primarily over the Magadha region and upheld political stability during a period of foreign threats, particularly from Indo-Greek rulers

Information Booster:

  • Pushyamitra Shunga initiated the Shunga dynasty after ending the Maurya rule
  • Their administration lasted from approximately 185 BCE to 73 BCE
  • Pataliputra continued to be the political center under the Shungas
  • The dynasty is noted for supporting Brahmanism and Sanskrit culture
  • Shunga rulers made significant contributions to Indian art and heritage
  • Agnimitra Shunga, son of Pushyamitra, was featured in Kalidasa’s drama “Malavikagnimitram”

Additional Knowledge:

  • Shaka dynasty: The Shakas, or Indo-Scythians, entered India in the 1st century BCE
  • Kushan dynasty: Originated from Central Asia; rose in the 1st century CE
  • Satavahana dynasty: Gained prominence slightly after the Shungas

Q.37 Who has been re-elected as the Speaker (Chairman) of the 18th Lok Sabha?

Options:

  • A. Jagdeep Dhankhar
  • B. Om Birla
  • C. Harivansh Narayan Singh
  • D. Sumitra Mahajan

Answer: B. Om Birla

Explanation: On 26 June 2024, Om Birla was re-elected as the Speaker of the 18th Lok Sabha. He was the candidate of the BJP-led NDA alliance and defeated the Congress candidate Kadikunnil Suresh. This election was the fourth Speaker election in the history of the Lok Sabha of India, which is usually held unanimously.

Key Points:

  • Om Birla has been serving as the Speaker of the Lok Sabha since 2019
  • This is his second term

Additional Information:

  • (A) Jagdeep Dhankhar – Currently the Chairman (Vice President) of the Rajya Sabha
  • (C) Harivansh Narayan Singh – Deputy Chairman of the Rajya Sabha
  • (D) Sumitra Mahajan – Was the Speaker of the 16th Lok Sabha

Q.38 The concept of “Equal Protection Under Law” in the Indian Constitution is borrowed from the Constitution of which of the following countries?

Options:

  • A. Australia
  • B. United Kingdom
  • C. Ireland
  • D. United States of America

Answer: D. United States of America

Explanation: The concept of “Equal Protection of the Laws” in the Indian Constitution is borrowed from the Constitution of the United States of America. This principle is enshrined in Article 14 of the Indian Constitution, which states that the State shall not deny to any person equality before the law or the equal protection of the laws within the territory of India.

Key Points:

  • “Equal Protection of the Laws” is a positive concept that implies the right to equal treatment in equal circumstances
  • “Like should be treated alike”

Information Booster:

  • Article 14 Dual Nature: Article 14 includes two distinct concepts: “Equality Before Law” (British origin) and “Equal Protection of the Laws” (American origin)
  • Positive vs. Negative Concept: “Equality Before Law” is a negative concept (absence of special privileges), while “Equal Protection of the Laws” is a positive concept
  • Fourteenth Amendment: The American concept originates from Section 1 of the 14th Amendment to the U.S. Constitution (1868)

Additional Knowledge:

  • Option (A) Australia: Major features borrowed include the Concurrent List, Freedom of Trade, Commerce and Intercourse
  • Option (B) United Kingdom: Borrowed Parliamentary form of government, Rule of Law, Single Citizenship, and concept of “Equality Before Law”
  • Option (C) Ireland: Borrowed the Directive Principles of State Policy (DPSP), method of election of the President

Q.39 At simple interest, an amount becomes Rs.1120 in 4 years and Rs.1200 in 5 years. The principal is:

Options:

  • A. Rs.800
  • B. Rs.1000
  • C. Rs.1050
  • D. Rs.1080

Answer: A. Rs.800

Explanation: Given:

  • Amount after 4 years = Rs.1120
  • Amount after 5 years = Rs.1200

Solution:

  • The difference between the two amounts is the simple interest for 1 year
  • Interest for 1 year = 1200 − 1120 = Rs.80
  • So, yearly interest = Rs.80
  • Interest for 4 years = 80 × 4 = Rs.320
  • Principal = Amount − Interest
  • Principal = 1120 − 320 = Rs.800

The correct answer is (A) Rs.800.

Q.40 ‘Black death’ is another name for

Options:

  • A. Kala-azar
  • B. Plague
  • C. Botulism
  • D. Tetanus

Answer: B. Plague

Explanation: The “Black Death” refers to the devastating pandemic of bubonic plague caused by Yersinia pestis in the 14th century. It caused widespread blackening of skin tissues due to hemorrhages, giving rise to the name. Historically, it led to massive mortality in Europe and remains one of the worst pandemics recorded.

Additional Information:

  • Option (A) Kala-azar is caused by Leishmania
  • Option (C) Botulism is caused by Clostridium botulinum
  • Option (D) Tetanus is caused by Clostridium tetani

Q.41 The wealth definition of economics was mentioned in _____________ by Adam Smith.

Options:

  • A. Capitalism, Socialism, and Democracy
  • B. Wealth of Nations
  • C. Principles of Economics
  • D. Nature and Significance of Economic Science

Answer: B. Wealth of Nations

Explanation: The wealth definition of economics was introduced by Adam Smith in his famous book “An Inquiry into the Nature and Causes of the Wealth of Nations”, published in 1776.

Key Points: According to this definition, economics is the study of how a nation acquires and increases its wealth, particularly focusing on production, distribution, and accumulation of wealth.

Adam Smith emphasized:

  • Material prosperity of nations
  • Wealth as the center of economic activity
  • Importance of free markets and labor productivity

Information Booster:

  • Capitalism, Socialism, and Democracy: Written by Joseph Schumpeter in 1942; discusses political economy
  • Principles of Economics: Written by Alfred Marshall in 1890; gave the welfare definition of economics
  • Nature and Significance of Economic Science: Written by Lionel Robbins in 1932; introduced the scarcity definition

Q.42 Amongst the following, who is known as the author of Arthashastra?

Options:

  • A. Kalidasa
  • B. Varahamihira
  • C. Aryabhatta
  • D. Kautilya

Answer: D. Kautilya

Explanation: Kautilya (also known as Chanakya or Vishnugupta) wrote the Arthashastra. It is an ancient Indian treatise on statecraft, economic policy, and military strategy. It served as the foundation for the Mauryan Empire’s administration.

Information Booster:

  • Kalidasa: Famous poet and playwright (Shakuntala, Meghaduta)
  • Aryabhatta: Great mathematician and astronomer (Aryabhatiya)
  • Varahamihira: Astronomer and mathematician (Pancha-Siddhantika)

Additional Knowledge:

  • The Arthashastra was rediscovered by R. Shamasastry in 1905
  • It is written in Sanskrit

Q.43 Who has been appointed as the new Governor of the Reserve Bank of India (RBI) on December 09, 2024?

Options:

  • A. Shaktikanta Das
  • B. Sanjay Malhotra
  • C. Urjit Patel
  • D. Raghuram Rajan

Answer: B. Sanjay Malhotra

Explanation: Sanjay Malhotra has been appointed as the new (26th) Governor of the Reserve Bank of India (RBI) on December 09, 2024. This appointment marks a significant leadership change at the RBI, which plays a crucial role in India’s monetary policy, financial stability, and economic growth.

Key Points:

  • His tenure starts on December 11, 2024 for a period of 3 years

Other Options:

  • Shaktikanta Das: Current RBI Governor since December 2018 (2nd longest serving RBI Governor)
  • Urjit Patel: Former RBI Governor (2016–2018)
  • Raghuram Rajan: Served as RBI Governor from 2013 to 2016

Key Facts about RBI:

  • Established on April 1, 1935, under the RBI Act, 1934
  • Responsible for issuing currency, regulating banks, and formulating monetary policies
  • The RBI Governor is appointed by the Government of India

Q.44 Who among the following has initially drafted the ‘Quit India’ resolution of the Indian National congress in 1942?

Options:

  • A. Mahatma Gandhi
  • B. Subhash Chandra Bose
  • C. Jayaprakash Narayan
  • D. BR Ambedkar

Answer: A. Mahatma Gandhi

Explanation: Mahatma Gandhi initially drafted the ‘Quit India’ resolution. During World War II, Mahatma Gandhi started the Quit India Movement, also known as the August Movement, during the Bombay session of the All India Congress Committee on August 8, 1942, demanding an end to British rule in India.

Information Booster:

  • Mumbai’s Gowalia Tank Maidan (also known as August Kranti Maidan) is the place where the quit India movement was launched by Mahatma Gandhi
  • He along with other leaders gathered here on August 8 and 9, 1942
  • The outcome was that Congress was declared an unlawful association
  • Its offices all over the country were raided
  • The leaders were arrested

Q.45 Which crop was widely cultivated by the Harappans besides wheat and barley?

Options:

  • A. Rice
  • B. Cotton
  • C. Sugarcane
  • D. Maize

Answer: B. Cotton

Explanation: The Harappans (Indus Valley Civilization) are believed to be the first people to cultivate Cotton. Pieces of woven cotton cloth were discovered at Mohenjodaro.

Information Booster:

  • The Greeks referred to cotton as ‘Sindon’, derived from the word ‘Sindh’, indicating its origin
  • Wheat and Barley were the main food crops, but cotton was a vital commercial crop

Additional Knowledge:

  • Rice (Option A): Evidence of rice cultivation is found in Lothal and Rangpur but was less common than wheat/barley
  • Sugarcane (Option C): Was not known to the Harappans

Q.46 National Emergency is declared under which Article of the Indian Constitution?

Options:

  • A. Article 352
  • B. Article 356
  • C. Article 360
  • D. Article 368

Answer: A. Article 352

Explanation: National Emergency is declared under Article 352 of the Indian Constitution. This provision allows the President of India to declare a national emergency if India’s security or any part of its territory is threatened by war, external aggression, or armed rebellion.

Grounds for Proclamation:

  • The President can issue a proclamation of emergency on the grounds of external aggression, war, or armed rebellion
  • Even if these events have not occurred but are imminent, allowing for preventive measures

Changes by the 44th Amendment Act:

  • The 44th Amendment Act of 1978 substituted the term “internal disturbance” with “armed rebellion”
  • This was done to remove vagueness and avoid misuse of the term

Approval of Parliament:

  • Once the President declares a national emergency, it must be approved by both Houses of Parliament within one month (reduced from two months by the 44th Amendment Act)

Duration and Extension:

  • The national emergency continues for six months after approval by Parliament and can be extended indefinitely with parliamentary approval every six months
  • This was a significant change introduced by the 44th Amendment Act

Information Booster:

  • Article 352 deals specifically with National Emergency in the Constitution
  • The 44th Amendment Act of 1978 was crucial in limiting the powers of the Executive
  • The President’s Proclamation must be approved by both Houses of Parliament within one month after being issued

Additional Knowledge:

  • Article 356 (Option B): Deals with President’s Rule (State Emergency)
  • Article 360 (Option C): Deals with the declaration of a Financial Emergency
  • Article 368 (Option D): Outlines the procedure for amending the Constitution

Q.47 Where was the first municipal corporation established in Bihar?

Options:

  • A. Patna
  • B. Gaya
  • C. Muzaffarpur
  • D. Motihari

Answer: A. Patna

Explanation: Patna was the first city in Bihar to have a municipal corporation established, playing an important role in urban governance and city management. The establishment helped formalize civic amenities, sanitation, and urban development.

Information Booster:

  • Municipal corporations in India oversee local administration
  • Including water supply, waste management, and infrastructure development

Q.48 Jarawa tribes can be found majorly in which part of India?

Options:

  • A. Kerala
  • B. Madhya Pradesh
  • C. Uttar Pradesh
  • D. Andaman & Nicobar Islands

Answer: D. Andaman & Nicobar Islands

Explanation: The Jarawa tribe is one of the five Particularly Vulnerable Tribal Groups (PVTGs) found in the Andaman & Nicobar Islands. They are an indigenous Negrito tribe that primarily inhabit the Middle and South Andaman Islands, living in a state of semi-isolation.

Information Booster:

  • They are protected under the Andaman and Nicobar Protection of Aboriginal Tribes Regulation (ANPATR), 1956
  • Contact with outsiders is restricted to prevent disease transmission and cultural disruption
  • Other PVTGs in Andaman include: Great Andamanese, Onge, Sentinelese, and Shompens

Additional Information:

  • Kerala: Home to Kurumba, Paniyan, Kattunayakan, and Irula tribes
  • Madhya Pradesh: Gond, Baiga, Bhil, and Korku being the major tribes
  • Uttar Pradesh: Primary tribes include Tharu, Bhoksa, and Buksa

Q.49 Who was elected as the Speaker of the 18th Bihar Legislative Assembly in December 2025?

Options:

  • A. Nand Kishore Yadav
  • B. Narendra Narayan Yadav
  • C. Dr. Prem Kumar
  • D. Samrat Chaudhary

Answer: C. Dr. Prem Kumar

Explanation: Following the 2025 Assembly Elections, senior BJP leader Dr. Prem Kumar was elected unanimously as the Speaker on December 2, 2025. The announcement was made by Narendra Narayan Yadav, who served as the Pro-tem Speaker to administer oaths to the 243 newly elected MLAs.

Key Points:

  • The election of Dr. Prem Kumar was unopposed as no other nominations were filed by the opposition
  • Nand Kishore Yadav was the Speaker of the previous (17th) Assembly

Information Booster:

  • Dr. Prem Kumar is a nine-term MLA from the Gaya Town constituency
  • Has served as a minister in multiple previous cabinets
  • The first to take oath in the 18th Assembly was Deputy CM Samrat Chaudhary, followed by Deputy CM Vijay Kumar Sinha
  • Maithili Thakur, the youngest MLA (Alinagar, Darbhanga), took her oath in Maithili while wearing the traditional paag

Q.50 When was the first-ever National Space Day (NSpD) observed across India?

Options:

  • A. 22 August 2023
  • B. 23 August 2023
  • C. 23 August 2024
  • D. 24 August 2024

Answer: C. 23 August 2024

Explanation: The first-ever National Space Day (NSpD) was observed across India on 23 August 2024. The day was commemorated to honor the Chandrayaan-3 mission’s successful landing on the Moon’s south pole on 23 August 2023. This historic achievement made India the first country to land a spacecraft near the lunar south pole.

Chandrayaan-3 Mission:

  • Launched by ISRO
  • India’s third lunar exploration mission
  • Aimed to land on the Moon’s south pole
  • Follow-up to Chandrayaan-2, focusing on landing and rover operations

Additional Information:

  • ISRO: Founded on 15 August 1969 by Dr. Vikram Sarabhai, often regarded as the father of the Indian space program
  • The organization was initially part of the Department of Atomic Energy
  • ISRO’s first satellite, Aryabhata, was launched on 19 April 1975
  • Headquarters: Bengaluru
  • Chairperson of ISRO: S. Somanath

Q.51 Who was the Viceroy of India at the time of civil disobedience movement?

Options:

  • A. Lord Wavell
  • B. Lord Reading
  • C. Lord Chelmsford
  • D. Lord Irwin

Answer: D. Lord Irwin

Explanation: Lord Irwin was the Viceroy of India from 1926 to 1931. The Civil Disobedience Movement began with the Dandi March in March 1930 during his tenure.

Information Booster:

  • The movement ended temporarily with the Gandhi-Irwin Pact in 1931
  • Simon Commission (1928) also visited India during Lord Irwin’s viceroyalty

Additional Knowledge:

  • Lord Wavell – Viceroy during the Cabinet Mission and Quit India Movement end
  • Lord Reading – Viceroy during the Chauri Chaura incident

Q.52 A shopkeeper bought 5 identical chairs for a total cost of Rs. 7,500. He sold three chairs at a gain of 20% each and the remaining two chairs at a loss of 10% each. What is his overall profit or loss percentage?

Options:

  • A. 8% Profit
  • B. 10% Profit
  • C. 12% Profit
  • D. 14% Profit

Answer: A. 8% Profit

Explanation: Given:

  • Total chairs = 5
  • Total Cost Price = 7500
  • Gain on 3 chairs = 20%
  • Loss on 2 chairs = 10%

Formula Used: Profit % = [(Total SP − Total CP) / Total CP] × 100

Solution:

  • Cost price per chair = 7500/5 = 1500
  • SP of one chair at 20% gain = 1500 × (120/100) = 1800
  • SP of 3 chairs = 3 × 1800 = 5400
  • SP of one chair at 10% loss = 1500 × (90/100) = 1350
  • SP of 2 chairs = 2 × 1350 = 2700
  • Total SP = 5400 + 2700 = 8100
  • Total CP = 7500
  • Profit = 8100 − 7500 = 600
  • Profit % = (600/7500) × 100 = 8%

Overall profit = 8%

Q.53 Who discovered the Cape of Good Hope?

Options:

  • A. Columbus
  • B. Vasco-da-Gama
  • C. Magellan
  • D. Bartholomew Diaz

Answer: D. Bartholomew Diaz

Explanation: In 1488, Portuguese explorer Bartholomew Diaz became the first European to sail around the southern tip of Africa. This region was originally called the “Cape of Storms” due to harsh weather. Later, King John II of Portugal renamed it the Cape of Good Hope because it opened up the possibility of reaching India and the East by sea.

Significance: The discovery of the Cape of Good Hope was a turning point in world history, as it allowed European nations to bypass the Middle East land routes and directly access the riches of Asia.

Information Booster:

  • Vasco da Gama later used the same route to reach Calicut (India) in 1498
  • Cape of Good Hope is located in South Africa today

Additional Information:

  • (A) Columbus: Reached America in 1492, searching for India
  • (B) Vasco-da-Gama: Used Diaz’s route to reach India in 1498
  • (C) Magellan: First circumnavigation of the Earth (1519–1522)

Q.54 Where will the Indian AI Research Organization (IAIRO) be established?

Options:

  • A. Bengaluru
  • B. Hyderabad
  • C. GIFT City, Gandhinagar
  • D. Pune

Answer: C. GIFT City, Gandhinagar

Explanation: The Government of Gujarat has approved the establishment of the Indian AI Research Organization (IAIRO) at GIFT City, Gandhinagar. IAIRO will become operational from January 1, 2026. It will be India’s first state-led AI research body, set up under a Public–Private Partnership (PPP) model.

Objective: The initiative aims to strengthen India’s AI ecosystem through collaboration between government, industry, and academia.

Information Booster:

  • Total outlay: ₹300 crore for the first five years
  • Funding pattern: Equal contribution (33.33% each) by:
    • Government of India
    • Government of Gujarat
    • Private partner (Indian Pharmaceutical Alliance – IPA)
  • Legal status: Non-profit organisation under Section 8 of the Companies Act, 2013, operating as an SPV
  • Technology model: Hybrid compute system with on-premise GPUs and integration with IndiaAI Cloud

Q.55 Where is the Tomb of Bakhtiyar Khan located in Bihar?

Options:

  • A. Rohtas
  • B. Kaimur
  • C. Nalanda
  • D. Patna

Answer: B. Kaimur

Explanation: Situated in the Kaimur district of Bihar, the Tomb of Bakhtiyar Khan stands as a significant monument of national importance. This architectural marvel exemplifies the fusion of Indo-Islamic styles.

Key Features:

  • The tomb is constructed in an octagonal shape
  • Elevated on a modest plinth
  • Gateway positioned on the eastern side
  • Adorned with verses from the Quran
  • Houses approximately thirty graves within its confines
  • Additional graves scattered across the outer courtyard

Q.56 Who among the following is the ex-officio Chairman of the Rajya Sabha?

Options:

  • A. Vice President of India
  • B. Finance Minister of India
  • C. President of India
  • D. Prime Minister of India

Answer: A. Vice President of India

Explanation: The Vice President of India serves as the ex-officio Chairman of the Rajya Sabha (the Council of States). “Ex-officio” means that by virtue of holding the office of the Vice President, the person automatically becomes the Chairman of the Rajya Sabha.

Constitutional Provisions:

  • This provision is mentioned in Article 64 and Article 89(1) of the Indian Constitution
  • While presiding over the House, the Chairman’s powers and functions are similar to those of the Speaker of the Lok Sabha
  • The Chairman is not a member of the House

Information Booster:

  • Salary and Functions: The Vice President does not draw a salary in the capacity of the Vice President; instead, they receive a salary and perquisites as the Chairman of the Rajya Sabha
  • Casting Vote: As the Chairman is not a member of the Rajya Sabha, they do not vote in the first instance
  • However, they can exercise a casting vote in the case of an equality of votes (a tie)
  • Acting President: When the Vice President acts as the President or discharges the functions of the President, they cease to perform the duties of the office of the Chairman of the Rajya Sabha

Additional Knowledge:

  • (B) Finance Minister: A member of the Union Cabinet but does not hold an ex-officio presiding role
  • (C) President of India: An integral part of the Parliament but not a member of either House
  • (D) Prime Minister of India: Leader of the House in the chamber to which they belong (usually Lok Sabha) but does not preside

Q.57 The 42nd Constitutional Amendment Act of Indian Constitution was passed in the year ________.

Options:

  • A. 1976
  • B. 1977
  • C. 1978
  • D. 1979

Answer: A. 1976

Explanation: The 42nd Constitutional Amendment Act was passed in 1976 during the emergency period under the leadership of Indira Gandhi. This amendment is often referred to as the “Mini Constitution” because it made several changes to the Constitution of India, including altering the Preamble.

Key Changes:

  • The amendment introduced changes in the Preamble to include the words “Socialist”, “Secular”, and “Integrity”
  • It enhanced the power of the central government while limiting the power of the states
  • The amendment also curtailed the power of the judiciary in matters of constitutional review
  • It placed restrictions on the right to move to the courts for the enforcement of fundamental rights
  • The Directive Principles of State Policy were given precedence over Fundamental Rights in certain cases

Q.58 Which of the following is NOT a cropping season in India?

Options:

  • A. Kharif
  • B. Rabi
  • C. Zaid
  • D. Plantation

Answer: D. Plantation

Explanation: India has three main cropping seasons based on climate and monsoon patterns:

  • Kharif crops are grown during the monsoon season from June to October
  • Rabi crops are grown during the winter season from October to March
  • Zaid crops are grown during the short summer season from March to June
  • Plantation is not a cropping season but a method of cultivation
  • Plantation crops are grown over many years rather than seasonal cycles

Information Booster:

  • Kharif crops depend mainly on the southwest monsoon
  • Rabi crops rely on winter rainfall and irrigation
  • Zaid crops require artificial irrigation and high temperatures
  • Major food grains are produced in Kharif and Rabi seasons
  • Seasonal cropping patterns are crucial for India’s food security

Additional Knowledge:

  • Kharif (Option A): Includes rice, maize, cotton, and millets
  • Rabi (Option B): Includes wheat, barley, mustard, and peas
  • Zaid (Option C): Includes watermelon, cucumber, vegetables, and fodder crops

Q.59 Match the following:

  1. Article 39 → 1. Equal pay for equal work B. Article 40 → 2. Organisation of village panchayats C. Article 51A → 3. Fundamental Duties D. Article 48A → 4. Environment protection

Options:

  • A. A-1, B-2, C-3, D-4
  • B. A-2, B-1, C-3, D-4
  • C. A-1, B-3, C-2, D-4
  • D. A-4, B-1, C-2, D-3

Answer: A. A-1, B-2, C-3, D-4

Explanation:

  • Article 39 (Directive Principles): Ensures equal pay for equal work for men and women → (1)
  • Article 40 (Directive Principles): Directs the state to organize village panchayats → (2)
  • Article 51A (Part IV-A): Contains Fundamental Duties of citizens → (3)
  • Article 48A (Directive Principles): Directs the state to protect and improve the environment → (4)

Information Booster:

  • DPSPs are non-justiciable but fundamental for governance
  • Fundamental Duties were added by the 42nd Constitutional Amendment Act, 1976
  • Panchayati Raj was given constitutional status by the 73rd Amendment Act, 1992
  • Environmental protection is also a Fundamental Duty under Article 51A(g)

Q.60 Match List-I with List-II: Central Trade Union Federation with Political Party

List-I (Central Trade Union Federation):

  • A. Centre of Indian Trade Unions
  • B. Indian National Trade Union Congress
  • C. Bharatiya Mazdoor Sangh
  • D. All India Trade Union Congress

List-II (Political Party):

    1. Indian National Congress
    1. Bharatiya Janata Party
    1. Communist Party of India
    1. Communist Party of India (Marxist)

Options:

  • A. A-4 B-1 C-2 D-3
  • B. A-4 B-2 C-1 D-3
  • C. A-3 B-2 C-1 D-4
  • D. A-3 B-1 C-2 D-4

Answer: A. A-4 B-1 C-2 D-3

Explanation: Here is the correct matching of trade unions with their affiliated political parties:

  • A. Centre of Indian Trade Unions (CITU) – (4) Communist Party of India (Marxist)
  • B. Indian National Trade Union Congress (INTUC) – (1) Indian National Congress
  • C. Bharatiya Mazdoor Sangh (BMS) – (2) Bharatiya Janata Party (via ideological link with RSS)
  • D. All India Trade Union Congress (AITUC) – (3) Communist Party of India

Information Booster:

  • These affiliations influence the policy positions and industrial actions taken by the respective unions
  • BMS is officially independent but ideologically aligned with Sangh Parivar
  • CITU and AITUC often collaborate in left-led labor movements
  • INTUC traditionally supports centrist labor reforms in line with Congress policies

Q.61 In Mendel’s experiment, what trait was used to determine the inheritance of pea plants?

Options:

  • A. Flower color
  • B. Seed shape
  • C. Plant height
  • D. All of the above

Answer: D. All of the above

Explanation: Mendel studied multiple traits, including flower color, seed shape, and plant height, to determine patterns of inheritance.

Information Booster:

  • Mendel selected traits with clear dominant and recessive forms
  • He focused on seven traits in pea plants, including seed color, seed shape, flower position, and pod shape
  • These traits were easy to distinguish and followed predictable inheritance patterns
  • Mendel’s experiments with these traits led to the discovery of his laws of inheritance
  • The controlled pollination allowed Mendel to track inheritance patterns

Q.62 The 73rd Constitutional Amendment Act of India is related to which of the following?

Options:

  • A. Urban local bodies
  • B. Panchayati Raj institutions
  • C. Cooperative societies
  • D. Scheduled Areas

Answer: B. Panchayati Raj institutions

Explanation: The 73rd Constitutional Amendment Act, 1992, gave constitutional status and protection to the Panchayati Raj institutions (PRIs).

Key Points:

  • It added a new Part IX to the Constitution entitled ‘The Panchayats’
  • It also added the 11th Schedule, which contains 29 functional items for Panchayats
  • The Act provides for a three-tier system of Panchayati Raj: Gram Panchayat (village), Panchayat Samiti (intermediate), and Zila Parishad (district)
  • It mandates regular elections every five years and reservation of seats for SCs, STs, and women

Information Booster:

  • April 24 is celebrated as National Panchayati Raj Day to commemorate this amendment
  • The amendment implemented the recommendation of the Balwant Rai Mehta Committee

Additional Knowledge:

  • Urban local bodies (Option A): Related to the 74th Constitutional Amendment Act
  • Cooperative societies (Option C): Related to the 97th Constitutional Amendment Act
  • Scheduled Areas (Option D): Primarily governed by the 5th and 6th Schedules

Q.63 The voting age was reduced from 21 years to 18 years by the ________ Constitutional Amendment Act of 1988.

Options:

  • A. 51st
  • B. 61st
  • C. 71st
  • D. 41st

Answer: B. 61st

Explanation: The 61st Constitutional Amendment Act, 1988 lowered the minimum voting age for Lok Sabha and State Legislative Assembly elections from 21 years to 18 years. This amendment was made to encourage youth participation in democracy and strengthen representative governance in India.

Information Booster:

  • Article Affected: Article 326, which deals with the right to vote in elections based on universal adult suffrage
  • Objective: To increase youth involvement in the electoral process and enhance political awareness
  • Introduced by: The then Prime Minister Rajiv Gandhi to promote greater participation of young voters

Additional Knowledge:

  • 51st Constitutional Amendment Act (1984): Provided special provisions for Scheduled Tribes
  • 71st Constitutional Amendment Act (1992): Included Konkani, Manipuri, and Nepali in the Eighth Schedule
  • 41st Constitutional Amendment Act (1976): Increased the retirement age of High Court judges

Q.64 What is the capital city of Venezuela?

Options:

  • A. Valencia
  • B. Maracaibo
  • C. Caracas
  • D. Barquisimeto

Answer: C. Caracas

Explanation: Caracas is the capital and largest city of Venezuela. It serves as the political, administrative, and cultural centre of the country.

Information Booster:

  • Located in a mountain valley close to the Caribbean coast
  • Houses major government institutions and the presidential palace

Additional Knowledge:

  • Maracaibo: An important oil city but not the capital
  • Currency: Bolivar

Q.65 In which Indian state has the State Election Commission initiated the rollout of a mobile-based e-voting system, making it the first to adopt such a system?

Options:

  • A. Uttar Pradesh
  • B. Bihar
  • C. Maharashtra
  • D. West Bengal

Answer: B. Bihar

Explanation: Bihar has become the first Indian state to roll out a mobile-based e-voting system. This move by the Bihar State Election Commission (SEC) is aimed at making the voting process more modern, accessible, and inclusive, especially for people who are unable to physically visit polling stations.

Key Points:

  • The initiative is part of Bihar’s efforts to embrace technological advancements in the electoral process

Information Booster:

  • This initiative is designed to enhance voter participation, particularly for people who face challenges in reaching physical polling booths
  • The mobile-based e-voting system is part of the State Election Commission’s efforts to modernize the voting process and increase accessibility
  • Bihar’s move could set a precedent for other states to consider adopting similar technologies

Additional Information:

  • E-Voting in India has been discussed as a way to improve voter turnout
  • While e-voting offers increased convenience, it raises concerns related to cybersecurity, voter privacy, and infrastructure requirements
  • Bihar is known for its proactive role in electoral reforms

Q.66 The Strait of Gibraltar separates which two of the following?

Options:

  • A. Atlantic ocean to Mediterranean sea
  • B. Mediterranean sea to Black sea
  • C. Chukchi sea to the arctic ocean
  • D. Beaufort sea to the East Siberian Sea

Answer: A. Atlantic ocean to Mediterranean sea

Explanation: The Strait of Gibraltar connects the Atlantic Ocean with the Mediterranean Sea and separates Spain on the European continent from Morocco on the African continent.

Q.67 Which ministry is the nodal ministry for implementing the Jal Jeevan Mission (JJM)?

Options:

  • A. Ministry of Rural Development
  • B. Ministry of Jal Shakti
  • C. Ministry of Environment, Forest and Climate Change
  • D. Ministry of Agriculture & Farmers Welfare

Answer: B. Ministry of Jal Shakti

Explanation: The Department of Drinking Water and Sanitation under the Ministry of Jal Shakti is the nodal authority for implementing the Jal Jeevan Mission. The mission focuses on providing safe and adequate drinking water to all rural households through Functional Household Tap Connections (FHTC).

Information Booster:

  • Ministry of Jal Shakti was formed in 2019 by merging the Ministry of Water Resources, River Development & Ganga Rejuvenation with the Ministry of Drinking Water & Sanitation
  • It oversees major schemes like Namami Gange, Atal Bhujal Yojana, and Jal Jeevan Mission
  • JJM type: Centrally Sponsored Scheme
  • Funding Pattern: 90:10 for Himalayan & NE States, 50:50 for other States, 100% for UTs

Q.68 When did Vasco da Gama, a Portuguese explorer, discover the sea route to India?

Options:

  • A. 1496
  • B. 1499
  • C. 1498
  • D. 1497

Answer: C. 1498

Explanation: Vasco da Gama discovered the sea route to India in 1498. He reached Calicut (Kozhikode) on the Malabar Coast on May 20, 1498. His voyage established the first direct maritime link between Europe and Asia. This discovery marked the beginning of Portuguese colonial influence in India.

Information Booster:

  • Vasco da Gama started his journey from Lisbon, Portugal in 1497
  • His route passed through the Cape of Good Hope (South Africa)
  • He was welcomed by the Zamorin (King) of Calicut
  • The discovery helped Portugal dominate the Indian Ocean trade in spices
  • Later, the Portuguese established trading centers at Goa, Diu, and Daman

Q.69 Who prepared and launched the draft of the Fifth Five-Year Plan?

Options:

  • A. Manmohan Singh
  • B. Jayaprakash Narayan
  • C. Bibek Debroy
  • D. D.P. Dhar

Answer: D. D.P. Dhar

Explanation: D.P. Dhar was the main architect of the Fifth Five-Year Plan (1974-1979). He was an economist and a key figure in India’s planning process during the time. The plan focused on self-reliance, agricultural growth, and addressing the economic challenges faced by India. It was launched during the tenure of the Prime Minister Indira Gandhi.

Additional Explanations:

  • Manmohan Singh: Later became the Prime Minister of India and played a significant role in India’s economic reforms in the 1990s, but was not responsible for the Fifth Five-Year Plan
  • Jayaprakash Narayan: Prominent social and political leader known for his role in the JP Movement, not involved in the Fifth Five-Year Plan
  • Bibek Debroy: Renowned economist and public policy expert, but not involved in the Fifth Five-Year Plan

Q.70 Translate the following English sentence into Hindi: “He did not agree with the decision taken by the committee.”

Options:

  • A. वह समिति द्वारा लिए गए निर्णय से सहमत नहीं है।
  • B. वह समिति के निर्णय से सहमत नहीं था।
  • C. समिति द्वारा लिया गया निर्णय उसे स्वीकार नहीं हुआ।
  • D. वह समिति द्वारा लिए गए निर्णय से सहमत नहीं था।

Answer: D. वह समिति द्वारा लिए गए निर्णय से सहमत नहीं था।

Explanation: The sentence is in the Simple Past Tense, as indicated by “did not agree.”

Detailed Explanation:

  • The auxiliary “did not” clearly shows a past-time negation
  • Option (D) correctly translates this past tense using “सहमत नहीं था”
  • The passive phrase “decision taken by the committee” is accurately rendered as “समिति द्वारा लिए गए निर्णय”
  • The sentence structure and emphasis are preserved correctly

Why Other Options Are Incorrect:

  • (A) uses present tense
  • (B) omits “by the committee,” making the translation incomplete
  • (C) changes the sentence construction and subject focus

Q.71 What is the chemical formula for glucose?

Options:

  • A. C₆H₁₂O₆
  • B. H₂O
  • C. CO₂
  • D. CH₄

Answer: A. C₆H₁₂O₆

Explanation: Glucose is a simple sugar (a monosaccharide) that is an important energy source in living organisms. Its chemical formula is C₆H₁₂O₆, indicating that each molecule contains 6 carbon atoms, 12 hydrogen atoms, and 6 oxygen atoms.

Additional Information:

  • (B) H₂O is the formula for water
  • (C) CO₂ is the formula for carbon dioxide
  • (D) CH₄ is the formula for methane

Q.72 Who was the primary founder of the Azad Hind Fauj (Indian National Army) in 1942?

Options:

  • A. Subhas Chandra Bose
  • B. Rash Behari Bose
  • C. Captain Mohan Singh
  • D. Bhagat Singh

Answer: C. Captain Mohan Singh

Explanation: The primary founder of the Azad Hind Fauj (Indian National Army) in 1942 was Captain Mohan Singh. Captain Mohan Singh, an officer in the British Indian Army, was the one who initially conceived and founded the Indian National Army (INA) in 1942 in Southeast Asia, with the help of Japanese forces, utilizing Indian Prisoners of War (POWs).

Information Booster:

  • Subhas Chandra Bose (Option A) later arrived in Southeast Asia in 1943, took over the leadership, and brilliantly reorganized, galvanized, and led the INA
  • He transformed it into a powerful military force for Indian independence
  • While he is the most famous figure associated with the INA, he was not the initial founder in 1942

Additional Knowledge:

  • Rash Behari Bose (Option B): Instrumental in founding the Indian Independence League and played a key role in the transfer of the INA’s leadership to Subhas Chandra Bose
  • Bhagat Singh (Option D): Revolutionary freedom fighter executed in 1931, well before the INA was formed

Q.73 Which Mughal emperor had a brother called Mirza Kamran, who conspired against him?

Options:

  • A. Babur
  • B. Akbar
  • C. Humayun
  • D. Jahangir

Answer: C. Humayun

Explanation: Humayun was the second Mughal emperor and the son of Babur. He had three brothers: Kamran, Askari, and Hindal. Mirza Kamran was the Governor of Kabul and Lahore and frequently rebelled or conspired against Humayun during his struggle to retain the Mughal throne.

Key Points:

  • Babur had advised Humayun to treat his brothers kindly, which Humayun followed to his own disadvantage for a long time

Information Booster:

  • Humayun was defeated by Sher Shah Suri in the battles of Chausa (1539) and Kanauj (1540), forcing him into exile
  • He eventually blinded Kamran Mirza after years of conflict to secure his position

Additional Knowledge:

  • Babur (Option A): Father of Humayun and founder of the Mughal Empire
  • Akbar (Option B): Son of Humayun; his half-brother Mirza Hakim also revolted against him
  • Jahangir (Option D): Son of Akbar; his own son Khusrau Mirza rebelled against him

Q.74 Arrange the following strategic straits of Indian Ocean from East to West.

  1. Palk strait B. Strait of Hormuz C. Bab-el-Mandep D. Malacca strait

Options:

  • A. D, A, B, C
  • B. A, B, C, D
  • C. D, C, B, A
  • D. C, A, D, B

Answer: A. D, A, B, C

Explanation: The strategic straits of the Indian Ocean play a crucial role in global trade and geopolitical dynamics. Here’s the correct order of these straits from east to west:

  1. Malacca Strait (D): Located between the Malay Peninsula (Malaysia) and Sumatra Island (Indonesia), the Malacca Strait is the easternmost strait. It’s a vital shipping lane connecting the Indian Ocean to the Pacific Ocean and carries a significant portion of global maritime trade.
  2. Palk Strait (A): Separating the southern tip of India from Sri Lanka, the Palk Strait lies west of the Malacca Strait. It serves as a maritime passage between the Bay of Bengal and the Arabian Sea.
  3. Strait of Hormuz (B): Situated at the mouth of the Persian Gulf, the Strait of Hormuz borders Iran and Oman. It’s the only outlet from the Persian Gulf to the Indian Ocean and a critical chokepoint for oil transportation.
  4. Bab-el-Mandeb Strait (C): Forming the southern entrance to the Red Sea, the Bab-el-Mandeb Strait separates Yemen on the Arabian Peninsula from Eritrea and Djibouti in Africa. It’s the westernmost strait on this list and connects the Red Sea to the Gulf of Aden and the Arabian Sea.

Q.75 What is the total literacy rate of Bihar according to the 2011 census of India?

Options:

  • A. 65.8 percent
  • B. 63.8 percent
  • C. 61.8 percent
  • D. 66.8 percent

Answer: C. 61.8 percent

Explanation: According to the 2011 Census of India, the total literacy rate of Bihar was 61.8%, which was one of the lowest literacy rates in the country at the time.

Information Booster:

  • The literacy rate in Bihar has seen gradual improvements, but it lagged behind the national average as of 2011
  • Efforts have been made through various government initiatives to improve education in the state
  • National average literacy rate (2011): 74.04%, with 82.14% of males and 65.46% of females literate

Q.76 Who was the first Chief Minister of Bihar?

Options:

  • A. Shri Krishna Singh
  • B. Satyapal Malik
  • C. Nitish Kumar
  • D. Rabri Devi

Answer: A. Shri Krishna Singh

Explanation: Shri Krishna Singh was the first Chief Minister of Bihar, serving from 1946 to 1961. He played a key role in the state’s development and was a prominent leader in Indian politics.

Information Booster:

  • He was a member of the Indian National Congress and worked closely with leaders like Jawaharlal Nehru
  • His tenure is remembered for various reforms in education, health, and infrastructure
  • He is known for his dedication to the welfare of Bihar’s rural areas

Additional Information:

  • (B) Satyapal Malik: A former Governor, not Bihar’s first Chief Minister
  • (C) Nitish Kumar: A later Chief Minister, not the first
  • (D) Rabri Devi: The first woman Chief Minister of Bihar but not the first overall

Q.77 Which of the following is the highest peak of Bihar?

Options:

  • A. Kaimur Hills
  • B. Barabar Hills
  • C. Someshwar Hills
  • D. Rajgir Hills

Answer: C. Someshwar Hills

Explanation: Someshwar Hills, located in the West Champaran district of Bihar, are part of the Siwalik range (Outer Himalayas). The highest peak of Bihar is Someshwar Peak, which lies in this hill range.

Key Points:

  • Located in North Western Bihar
  • It reaches a height of approximately 880 meters (2,887 feet) above sea level
  • Makes it the tallest natural elevation point in Bihar

Q.78 What is the Hindi meaning of the word “Transfer”?

Options:

  • A. स्वागत
  • B. स्थानांतरण
  • C. सूचना
  • D. विश्राम

Answer: B. स्थानांतरण

Explanation: Correct Option: B) स्थानांतरण (Sthanantaran)

This is the formal Hindi word for “Transfer.” It is derived from Sthan (place) and Antaran (movement/change), meaning to move from one place to another.

Other Options:

  • स्वागत (Swagat): Means “Welcome”
  • सूचना (Suchna): Means “Information” or “Notice”
  • विश्राम (Vishram): Means “Rest”

Usage Example:

  • English: He got a transfer to the Delhi office
  • Hindi: उनका दिल्ली कार्यालय में स्थानांतरण हो गया है।

Q.79 Which river is responsible for forming a major delta along the Coromandel Coastal Plain?

Options:

  • A. Godavari
  • B. Yamuna
  • C. Narmada
  • D. Brahmaputra

Answer: A. Godavari

Explanation: The Godavari River forms a prominent delta along the Coromandel Coastal Plain. It flows eastward and drains into the Bay of Bengal, depositing large quantities of alluvium near its mouth. Continuous sediment deposition over time has led to the formation of a fertile and extensive delta. This delta region supports intensive agriculture, especially rice cultivation.

Key Points:

  • The Coromandel Coast receives several east-flowing rivers, but the Godavari forms one of the largest deltas

Information Booster:

  • The Godavari River is the second-longest river in India after the Ganga
  • Flows eastward from the Western Ghats (Trimbakeshwar, Maharashtra) to the Bay of Bengal
  • Drains a large basin across central and southern India
  • Often referred to as the “Dakshin Ganga” (Ganga of the South) because of its religious importance, extensive basin, and role in supporting agriculture

About Deltaic Regions:

  • Characterized by fertile alluvial soils, abundant water supply, and intensive agriculture
  • These favorable conditions support high population density and productive farming, especially of crops like rice

Additional Information:

  • (B) Yamuna: Major tributary of the Ganga flowing through northern India; does not reach the Coromandel Coast
  • (C) Narmada: A west-flowing river that drains into the Arabian Sea and forms an estuary
  • (D) Brahmaputra: Forms a vast delta with the Ganga in eastern India (Sundarbans region), not on the Coromandel Coast

Q.80 On June 16, 2025, Prime Minister Narendra Modi was conferred with which of the following highest civilian honours by Cyprus?

Options:

  • A. Grand Collar of the Order of Makarios III
  • B. Grand Cross of the Order of Makarios III
  • C. Grand Commander of the Order of Makarios III
  • D. Knight Commander of the Order of Makarios III

Answer: B. Grand Cross of the Order of Makarios III

Explanation: On June 16, 2025, during his official visit to Cyprus, Prime Minister Narendra Modi was awarded the Grand Cross of the Order of Makarios III, Cyprus’ highest civilian honour. This prestigious award recognizes leaders who have made outstanding contributions to international relations and global cooperation.

Key Points:

  • PM Modi’s efforts to strengthen India-Cyprus relations, including promoting innovation, energy cooperation, and technology collaboration, were key factors in receiving this honour
  • The award is named after Archbishop Makarios III, the first President of Cyprus
  • Modi’s visit marked a historic milestone, as he became the first Indian Prime Minister to visit Cyprus

Information Booster:

  • The Order of Makarios III is named after the first President of Cyprus, Archbishop Makarios III
  • The award has several categories, and PM Modi received the Grand Cross
  • This is the first visit of an Indian Prime Minister to Cyprus
  • The award is conferred to dignitaries who contribute to global peace and diplomatic relations
  • Cyprus aims to expand strategic and economic ties with India through this recognition

Additional Information:

  • (A) The Grand Collar is a higher category than the Grand Cross, typically reserved for the most distinguished individuals
  • (C) The Grand Commander is another level of distinction but not the highest
  • (D) Knight Commander is a lower rank within the Order of Makarios III

Q.81 Which of the following is not caused by the atmospheric refraction of light?

Options:

  • A. Twinkling of stars at night
  • B. Sun appearing higher in the sky than it actually is
  • C. Sun becoming visible two minutes before actual sunrise
  • D. Sun appearing red at sunset

Answer: D. Sun appearing red at sunset

Explanation: The red appearance of the sun at sunset is caused by scattering of shorter wavelengths of light by atmospheric particles. Atmospheric refraction is responsible for apparent position changes, not colour change.

Additional Information:

  • Option (A) is incorrect because twinkling is due to refraction
  • Option (B) is incorrect because refraction raises the apparent position of the sun
  • Option (C) is incorrect because early sunrise is caused by refraction

Q.82 One grain is sold for ₹84/kg at 20% profit. Another is sold for ₹72/kg at 20% profit. Mixed in ratio 3:2 and sold at ₹78/kg, what is the profit/loss percentage?

Options:

  • A. 10% profit
  • B. 15.15% loss
  • C. 18.18% profit
  • D. 20% profit

Answer: C. 18.18% profit

Explanation: Given:

  • SP of first grain = 84
  • Profit = 20%
  • SP of second grain = 72
  • Profit = 20%
  • Ratio = 3 : 2
  • SP of mixture = 78

Formula Used:

  • CP = (SP × 100) / (100 + Profit%)
  • Profit % = [(SP − CP) / CP] × 100

Solution:

  • CP₁ = (84 × 100) / 120 = 70
  • CP₂ = (72 × 100) / 120 = 60
  • Mean CP = (3 × 70 + 2 × 60) / 5 = 330 / 5 = 66
  • Profit = 78 − 66 = 12
  • Profit % = (12 / 66) × 100 = 18.18%

Final Answer: 18.18%

Q.83 Consider the following statements:

  1. The Parliamentary system is borrowed from Britain.
  2. The concept of Directive Principles is from Ireland.
  3. The idea of Fundamental Duties is taken from USSR.

Options:

  • A. 1 and 2 only
  • B. 2 and 3 only
  • C. 1 and 3 only
  • D. All of the above

Answer: D. All of the above

Explanation: All three statements are correct and reflect the constitutional borrowings made by India in 1949. These provisions were adopted to strengthen democracy, welfare orientation, and citizens’ responsibilities.

Information Booster:

  • Britain → Parliamentary system, Rule of Law, Single Citizenship
  • Ireland → DPSPs, Nomination of Rajya Sabha members
  • USSR (Russia) → Fundamental Duties, Five-Year Plans
  • USA → Fundamental Rights, Judicial Review
  • Canada → Federal system with a strong Centre

Additional Knowledge:

  • Fundamental Duties were added by the 42nd Constitutional Amendment Act, 1976
  • Directive Principles are listed in Part IV (Articles 36–51)
  • Parliamentary system ensures collective responsibility and real executive authority in the Council of Ministers

Q.84 What is the Hindi meaning of the word “Plan”?

Options:

  • A. योजना
  • B. फल
  • C. नगर
  • D. समय

Answer: A. योजना

Explanation: Correct Option: A) योजना (Yojana)

This is the direct translation for “Plan.” It refers to a detailed proposal for doing or achieving something.

Other Options:

  • फल (Phal): Means “Fruit” or “Result”
  • नगर (Nagar): Means “City” or “Town”
  • समय (Samay): Means “Time”

Usage Example:

  • English: We need a new plan for this project
  • Hindi: हमें इस परियोजना के लिए एक नई योजना की आवश्यकता है।

Q.85 In which year was Delhi officially announced as the Capital of British India by then Emperor George V?

Options:

  • A. 1911
  • B. 1907
  • C. 1913
  • D. 1910

Answer: A. 1911

Explanation: In December 1911, during the Delhi Durbar, Emperor George V of Britain made the historic announcement that the capital of British India would be shifted from Calcutta (now Kolkata) to Delhi.

Key Points:

  • This decision was influenced by Delhi’s historical significance and its strategic location
  • The announcement marked a significant shift in the British administration’s focus and led to the development of New Delhi, designed by British architects Sir Edwin Lutyens and Sir Herbert Baker

Q.86 Champaran Satyagraha was led by whom among the following freedom fighters of British India?

Options:

  • A. Lal Bahadur Shastri
  • B. Rajendra Prasad
  • C. Mahatma Gandhi
  • D. Subhash Chandra Bose

Answer: C. Mahatma Gandhi

Explanation: Champaran Satyagraha (1917) was Mahatma Gandhi’s first Satyagraha in India. It was launched in Champaran, Bihar, to protest against the exploitation of indigo farmers by British planters. The movement marked the beginning of Gandhi’s leadership in India’s freedom struggle.

Key Points:

  • Issue: Forced Indigo cultivation by British planters
  • Outcome: British abolished the Tinkathia system (compulsory indigo farming)

Additional Information:

  • Lal Bahadur Shastri: Gave the slogan “Jai Jawan Jai Kisan”
  • Rajendra Prasad: Assisted Gandhi in Champaran, later became India’s first President
  • Subhash Chandra Bose: Led Azad Hind Fauj, known for “Give me blood, and I will give you freedom”

Q.87 Which river forms an estuary before merging into the Arabian Sea?

Options:

  • A. Narmada
  • B. Mahanadi
  • C. Godavari
  • D. Ganga

Answer: A. Narmada

Explanation: The Narmada River is the river that forms an estuary before merging into the Arabian Sea. The river passes through the states Madhya Pradesh and Gujarat.

About Estuaries: An estuary is a coastal area where freshwater from a river meets and mixes with saltwater from the sea, supporting a variety of marine and freshwater ecosystems.

Information Booster:

  • The Narmada River forms an estuary before merging into the Arabian Sea
  • The Narmada flows westward and is the largest river in India that drains into the Arabian Sea
  • Other major Indian rivers, such as the Ganga, Godavari, and Mahanadi, empty into the Bay of Bengal rather than the Arabian Sea
  • The Narmada River originates from the Amarkantak Plateau in Madhya Pradesh

Q.88 Which statement correctly describes the presence of jet streams over India during the monsoon season?

Options:

  • A. Only the easterly jet stream is present
  • B. Both westerly and easterly jet streams are present
  • C. Both westerly and easterly jet streams disappear
  • D. Only the westerly jet stream is present

Answer: B. Both westerly and easterly jet streams are present

Explanation: During the Indian monsoon season, both the subtropical westerly jet stream and the tropical easterly jet stream influence the Indian region. The easterly jet stream develops over peninsular India during summer and plays a crucial role in strengthening the southwest monsoon. The westerly jet stream shifts northward but does not completely disappear. The interaction of these jet streams affects rainfall distribution and monsoon intensity.

Information Booster:

  • Jet streams are narrow, fast-flowing air currents located in the upper troposphere, generally between 9–16 km above the Earth’s surface
  • They flow from west to east or east to west with very high wind speeds
  • They play a key role in atmospheric circulation
  • The tropical easterly jet (TEJ) develops over the Indian subcontinent during summer due to intense heating of the Tibetan Plateau
  • Its presence is closely associated with strong and well-distributed southwest monsoon rainfall over India

Q.89 Which group among the following consists exclusively of sedimentary rocks?

Options:

  • A. Limestone, bauxite and non-foliated rocks
  • B. Non-foliated rocks, granite and shale
  • C. Sandstone, limestone and shale
  • D. Sandstone, bauxite and foliated rocks

Answer: C. Sandstone, limestone and shale

Explanation: Sedimentary rocks are formed through the deposition, compaction, and cementation of sediments.

Key Points:

  • Sandstone: A clastic sedimentary rock formed from sand-sized particles
  • Limestone: A sedimentary rock mainly composed of calcium carbonate, often formed from marine organisms
  • Shale: A fine-grained sedimentary rock formed from clay and silt particles
  • All three rocks listed in option (C) belong to the sedimentary category

Information Booster:

  • Sedimentary rocks are typically layered (stratified) in structure
  • Often contain fossils, as they are formed by the deposition and compaction of sediments under relatively low temperature and pressure conditions
  • Cover the largest area of the Earth’s surface among the three major rock types
  • Especially form extensive plains, plateaus, and continental shelves, making them highly significant in physical geography

Economic Importance:

  • Sedimentary rocks are the main sources of coal, petroleum, natural gas, limestone, sandstone, and gypsum
  • These are widely used as fuel resources and building materials

Q.90 Rs. 15,000 is invested at the rate of 10% p.a. for 3 years at simple interest in scheme A. The amount received is then invested for 2 years at 5% p.a., compounded annually in scheme B. Find the final amount.

Options:

  • A. Rs. 21498.75
  • B. Rs. 19,845.65
  • C. Rs. 20,250.45
  • D. Rs. 19,500.25

Answer: A. Rs. 21498.75

Explanation: Given:

  • P = 15000
  • R₁ = 10%
  • T₁ = 3 years
  • R₂ = 5%
  • T₂ = 2 years

Formula Used:

  • A = P (1 + RT/100) [for simple interest]
  • A = P (1 + R/100)^T [for compound interest]

Solution:

  • A₁ = 15000 (1 + 10 × 3/100)
  • A₁ = 15000 × 1.30 = 19500
  • A₂ = 19500 (1 + 5/100)²
  • A₂ = 19500 × 1.1025
  • A₂ = 21498.75

Final Answer: 21498.75

Q.91 Who was elected as Ireland’s 10th President in October 2025?

Options:

  • A. Heather Humphreys
  • B. Catherine Connolly
  • C. Mary McAleese
  • D. Michael D. Higgins

Answer: B. Catherine Connolly

Explanation: Catherine Connolly, an independent leftwing politician from Galway, won the Irish presidential election 2025 with approximately 63% of first-preference votes. She becomes Ireland’s 10th President and the third woman to hold the office.

Information Booster:

  • Election held on 24 October 2025
  • Main rival: Heather Humphreys, who received ~29% votes
  • Turnout: ~46%, with a record number of spoiled ballots
  • Connolly emphasizes social justice, inclusivity, and Ireland’s neutrality

Additional Knowledge:

  • Previously served as Teachta Dála (TD) for Galway West since 2016
  • Held position of Deputy Speaker (LeasCeann Comhairle) in Irish Parliament
  • Advocates policies on housing, health, and opportunity equality
  • Her victory reflects shift in voter sentiment against centreright establishment

Q.92 Who discovered Penicillin?

Options:

  • A. Charles Darwin
  • B. Alexander Fleming
  • C. Louis Pasteur
  • D. Gregor Mendel

Answer: B. Alexander Fleming

Explanation: Alexander Fleming discovered Penicillin in 1928, revolutionizing medicine by introducing the first antibiotic.

About Penicillin:

  • Discovered by Alexander Fleming in 1928
  • Played a major role in treating bacterial infections
  • Nobel Prize in 1945 awarded to Fleming, along with Howard Florey and Ernst Chain for the development of Penicillin as an effective antibiotic

Additional Information:

  • Charles Darwin: Known for the theory of evolution
  • Louis Pasteur: Developed germ theory and vaccines
  • Gregor Mendel: Known for genetics and inheritance laws

Q.93 Who became India’s 90th Chess Grandmaster after achieving his final GM norm at the Bijeljina Open in Bosnia?

Options:

  • A. R Praggnanandhaa
  • B. Ilamparthi AR
  • C. D Gukesh
  • D. Nihal Sarin

Answer: B. Ilamparthi AR

Explanation: Ilamparthi AR, a 16-year-old chess prodigy from Chennai, Tamil Nadu, became India’s 90th Chess Grandmaster after securing his final GM norm at the Bijeljina Open in Bosnia.

Key Achievements:

  • Earlier norms at the Ha Noi Tournament (Vietnam, 2023) and Singapore International Open (2024)
  • Crossed the 2500 Elo rating mark during the Rilton Cup (2024–25)
  • Trained by Grandmaster Shyam Sundar M
  • Studies at Velammal School, known for producing top chess players

Information Booster:

  • Age: 16 years
  • Home State: Tamil Nadu
  • Final GM Norm: Bijeljina Open, Bosnia
  • Coach: GM Shyam Sundar M

Additional Knowledge:

  • Ilamparthi was supported early in his career by Dutch GM Anish Giri
  • Tamil Nadu has produced India’s top chess talents such as Viswanathan Anand, D Gukesh, and R Praggnanandhaa
  • Grandmaster (GM) is the highest title awarded by the World Chess Federation (FIDE)
  • His achievement strengthens Tamil Nadu’s reputation as the chess powerhouse of India

Q.94 Kanishka’s capital was at

Options:

  • A. Mathura
  • B. Peshawar
  • C. Amravati
  • D. Kanauj

Answer: B. Peshawar

Explanation: Kanishka came to rule an empire in Bactria extending from Turfan in the Tarim Basin to Pataliputra on the Gangetic plain. The main capital of his empire was located at Purushapura, modern day Peshawar.

Q.95 Consider the following statements:

  1. Money Bills can only be introduced in the Lok Sabha.
  2. Rajya Sabha has power to reject a Money Bill.
  3. Speaker of Lok Sabha decides whether a bill is a Money Bill.

Options:

  • A. 1 and 2 only
  • B. 1 and 3 only
  • C. 2 and 3 only
  • D. All of the above

Answer: B. 1 and 3 only

Explanation:

  • Money Bills can be introduced only in the Lok Sabha under Article 110 of the Constitution
  • The Speaker of the Lok Sabha certifies whether a Bill is a Money Bill, and the Speaker’s decision is final
  • The Rajya Sabha cannot reject or amend a Money Bill; it can only recommend changes, which the Lok Sabha may accept or reject

Information Booster:

  • A Money Bill deals only with taxes, borrowing, expenditure, or the Consolidated Fund of India
  • It must be passed by Lok Sabha and sent to Rajya Sabha for recommendations (within 14 days)
  • If Rajya Sabha does not return the Bill within 14 days, it is deemed passed
  • The President can only recommend, not return, a Money Bill
  • Example: Finance Bill, Appropriation Bill

Additional Knowledge:

  • Article 109 – Procedure in Rajya Sabha for Money Bills
  • Article 110 – Definition of Money Bill
  • Article 117 – Special provisions for Financial Bills
  • First Speaker of Lok Sabha: G.V. Mavalankar
  • Union Budget is also presented only in Lok Sabha

Q.96 Which of the following states has mangrove forests?

Options:

  • A. Telangana
  • B. Andhra Pradesh
  • C. Rajasthan
  • D. Manipur

Answer: B. Andhra Pradesh

Explanation: Andhra Pradesh has mangrove forests. Andhra Pradesh has 582 square kilometers of mangrove forests, which is about 9% of the state’s local forest area.

Mangrove Forests in Andhra Pradesh:

  • Coringa Wildlife Sanctuary: This estuary near Kakinada is India’s third largest mangrove forest stretch
  • Has 24 species of mangrove trees and over 120 bird species

Other States with Mangrove Forests:

  • West Bengal: Has the highest share of mangrove forests in India, with over 42% in 2021; the Sundarbans national park is located in West Bengal
  • Gujarat: Has 23.66% of India’s mangrove cover
  • Andaman & Nicobar Islands: Has 12.39% of India’s mangrove cover

About Mangroves:

  • Represent a characteristic littoral (near the seashore) forest ecosystem
  • Mostly evergreen forests that grow in sheltered lowlying coasts, estuaries, mudflats, tidal creeks, backwaters (coastal waters held back on land), marshes, and lagoons of tropical and subtropical regions
  • Grow below the high water level of spring tides

Q.97 The Gandak River is a tributary of which river?

Options:

  • A. Yamuna River
  • B. Ganga River
  • C. Kaveri River
  • D. Brahmaputra River

Answer: B. Ganga River

Explanation: The Gandak River is a major tributary of the Ganga River. It originates from Nepal, flows through Bihar, and finally joins the Ganga River near Hajipur in Bihar. It is known for its high sediment load and is considered one of the fastest-flowing rivers in India.

About Gandak River:

  • Source & Course:
    • Originates from Nhubine Himal Glacier, Nepal
    • Flows through Nepal and India (Uttar Pradesh & Bihar)
    • Merges with Ganga River near Sonepur, Bihar
  • Major Tributaries:
    • Right Bank: Trishuli, Budhi Gandak
    • Left Bank: Mayangadi, Danda, Panchnad

About Ganga River:

  • Origin: Gangotri Glacier, Uttarakhand (as Bhagirathi)
  • Length: 2,525 km (Longest river in India)
  • Mouth: Bay of Bengal
  • States Covered: Uttarakhand, Uttar Pradesh, Bihar, Jharkhand, West Bengal
  • Left-Bank Tributaries: Ramganga, Gomti, Ghaghara, Gandak, Kosi, Mahananda
  • Right-Bank Tributaries: Yamuna, Tamsa (Tons), Son, Punpun, Damodar

Q.98 When was NITI Aayog established?

Options:

  • A. 2015
  • B. 2016
  • C. 2017
  • D. 2018

Answer: A. 2015

Explanation: NITI Aayog (National Institution for Transforming India) was established on January 1, 2015, replacing the Planning Commission. It serves as the policy think tank of the Government of India, aimed at fostering cooperative federalism and ensuring sustainable development through strategic planning and innovation.

Key Objectives of NITI Aayog:

  • Promote cooperative federalism through structured support initiatives
  • Formulate policies and programs to achieve sustainable development goals (SDGs)
  • Foster innovation and leverage technology to enhance governance
  • Act as a platform for resolution of inter-sectoral and inter-departmental issues

Additional Information:

  • Chairperson: The Prime Minister of India
  • Vice Chairperson: Appointed by the Prime Minister
  • Full-Time Members: Includes domain experts and economists
  • NITI Aayog plays a pivotal role in driving India’s economic and social transformation

Q.99 The cell wall of bacteria is made up of:

Options:

  • A. peptidoglycan
  • B. cellulose
  • C. glycogen
  • D. peptone

Answer: A. peptidoglycan

Explanation: The cell wall of bacteria is primarily composed of a substance called peptidoglycan. This polymer is made up of sugars and amino acids that form a rigid structure, which helps maintain the shape of the bacterial cell and protects it from external pressures.

Key Points:

  • Peptidoglycan is unique to bacteria and is not found in the cells of plants, animals, or fungi
  • Peptidoglycan is a critical component for bacterial survival, as it provides structural support
  • It is a target for certain antibiotics like penicillin that inhibit its synthesis, weakening the bacterial cell wall
  • The structure of the bacterial cell wall can vary between different groups of bacteria
  • Gram-positive bacteria have a thick peptidoglycan layer
  • Gram-negative bacteria have a thinner layer with an outer membrane

Other Options:

  • Cellulose: Found in the cell walls of plants, not bacteria
  • Glycogen: A form of stored carbohydrate in animals and fungi, not a component of bacterial cell walls
  • Peptone: A mixture of peptides and amino acids derived from proteins, often used as a nutrient source in microbiological media, but not a part of the bacterial cell wall

Q.100 Kanger Valley National Park is located in which district of Chhattisgarh?

Options:

  • A. Dantewada
  • B. Bastar
  • C. Kanker
  • D. Bijapur

Answer: B. Bastar

Explanation: Kanger Valley National Park is located in the Bastar district of Chhattisgarh, near the city of Jagdalpur. It derives its name from the Kanger River, which flows centrally through the park.

Recent Recognition: In early 2025, the park was added to the UNESCO World Heritage Tentative List under the ‘Natural’ category, recognizing its unique biological and geological diversity.

Information Booster:

  • Geological Wonder: The park is most famous for its subterranean limestone caves
  • Notable Caves:
    • Kotumsar Cave: Known for its blind fish
    • Kailash Cave
    • Dandak Cave
  • These caves feature spectacular stalactite and stalagmite formations
  • Biodiversity: One of the densest pockets of biodiversity in Central India
  • Primary habitat for the Bastar Hill Myna (the State Bird of Chhattisgarh)

How to Calculate Marks Using Bihar Police SI Answer Key 2026?

Marking Scheme:

  • +2 marks for each correct answer

  • –0.2 marks for each wrong answer

  • 0 marks for unattempted

Formula:

Score = (Correct Answers × 2) – (Wrong Answers × 0.2)

Sharing is caring!

[banner_management slug=bihar-police-si-answer-key-2026]

TOPICS: